Why should K be closed to ensure X/K is complete?










3












$begingroup$


If $K$ is a closed subspace of Banach space $X$, then $X/K$ is complete. But I think the usual proof of this theorem doesn't make use of the fact that $K$ is closed. Would anyone explain it to me? Thanks a lot.










share|cite|improve this question









$endgroup$
















    3












    $begingroup$


    If $K$ is a closed subspace of Banach space $X$, then $X/K$ is complete. But I think the usual proof of this theorem doesn't make use of the fact that $K$ is closed. Would anyone explain it to me? Thanks a lot.










    share|cite|improve this question









    $endgroup$














      3












      3








      3





      $begingroup$


      If $K$ is a closed subspace of Banach space $X$, then $X/K$ is complete. But I think the usual proof of this theorem doesn't make use of the fact that $K$ is closed. Would anyone explain it to me? Thanks a lot.










      share|cite|improve this question









      $endgroup$




      If $K$ is a closed subspace of Banach space $X$, then $X/K$ is complete. But I think the usual proof of this theorem doesn't make use of the fact that $K$ is closed. Would anyone explain it to me? Thanks a lot.







      functional-analysis






      share|cite|improve this question













      share|cite|improve this question











      share|cite|improve this question




      share|cite|improve this question










      asked Nov 11 '18 at 18:29









      ZengZeng

      524




      524




















          3 Answers
          3






          active

          oldest

          votes


















          7












          $begingroup$

          If the subspace $K$ is not closed, then the quotient $X/K$ is not even Hausdorff, so does not meet the usual requirements of a topological vector space at all!



          (It's not about completeness or not, as $X/K$ is not metric in that case!)



          EDIT: In more detail, for $K$ not closed, and for $x$ in the closure of $K$ but not in $K$ itself, every neighborhood of $0$ in the quotient (with the quotient topology) contains $x$, but $xnot=0$ in the quotient. So the quotient would be non-Hausdorff (since two distinct points, $0$ and $x$, do not have disjoint neighborhoods).



          A Banach space, or even a normed space, is Hausdorff, as is every metric space.






          share|cite|improve this answer











          $endgroup$












          • $begingroup$
            Thanks! But it has been a long time since my last topology class, so would you give me some hint?
            $endgroup$
            – Zeng
            Nov 11 '18 at 18:39


















          4












          $begingroup$

          If $K$ is not closed, then the function $lVert x+KrVert=inf_yin K lVert x-yrVert$ is not a norm on the quotient space, but just a seminorm, because $lVert x+KrVert=0$ for all $xinoverline K$.






          share|cite|improve this answer









          $endgroup$




















            2












            $begingroup$

            You do need closure of $K$. In particular, you need it to show that $X/K$ is in fact a normed space: If $| x + K |=0$, closure of $K$ implies that $x+K=K=0_X/K$.






            share|cite|improve this answer









            $endgroup$












              Your Answer





              StackExchange.ifUsing("editor", function ()
              return StackExchange.using("mathjaxEditing", function ()
              StackExchange.MarkdownEditor.creationCallbacks.add(function (editor, postfix)
              StackExchange.mathjaxEditing.prepareWmdForMathJax(editor, postfix, [["$", "$"], ["\\(","\\)"]]);
              );
              );
              , "mathjax-editing");

              StackExchange.ready(function()
              var channelOptions =
              tags: "".split(" "),
              id: "69"
              ;
              initTagRenderer("".split(" "), "".split(" "), channelOptions);

              StackExchange.using("externalEditor", function()
              // Have to fire editor after snippets, if snippets enabled
              if (StackExchange.settings.snippets.snippetsEnabled)
              StackExchange.using("snippets", function()
              createEditor();
              );

              else
              createEditor();

              );

              function createEditor()
              StackExchange.prepareEditor(
              heartbeatType: 'answer',
              autoActivateHeartbeat: false,
              convertImagesToLinks: true,
              noModals: true,
              showLowRepImageUploadWarning: true,
              reputationToPostImages: 10,
              bindNavPrevention: true,
              postfix: "",
              imageUploader:
              brandingHtml: "Powered by u003ca class="icon-imgur-white" href="https://imgur.com/"u003eu003c/au003e",
              contentPolicyHtml: "User contributions licensed under u003ca href="https://creativecommons.org/licenses/by-sa/3.0/"u003ecc by-sa 3.0 with attribution requiredu003c/au003e u003ca href="https://stackoverflow.com/legal/content-policy"u003e(content policy)u003c/au003e",
              allowUrls: true
              ,
              noCode: true, onDemand: true,
              discardSelector: ".discard-answer"
              ,immediatelyShowMarkdownHelp:true
              );



              );













              draft saved

              draft discarded


















              StackExchange.ready(
              function ()
              StackExchange.openid.initPostLogin('.new-post-login', 'https%3a%2f%2fmath.stackexchange.com%2fquestions%2f2994221%2fwhy-should-k-be-closed-to-ensure-x-k-is-complete%23new-answer', 'question_page');

              );

              Post as a guest















              Required, but never shown

























              3 Answers
              3






              active

              oldest

              votes








              3 Answers
              3






              active

              oldest

              votes









              active

              oldest

              votes






              active

              oldest

              votes









              7












              $begingroup$

              If the subspace $K$ is not closed, then the quotient $X/K$ is not even Hausdorff, so does not meet the usual requirements of a topological vector space at all!



              (It's not about completeness or not, as $X/K$ is not metric in that case!)



              EDIT: In more detail, for $K$ not closed, and for $x$ in the closure of $K$ but not in $K$ itself, every neighborhood of $0$ in the quotient (with the quotient topology) contains $x$, but $xnot=0$ in the quotient. So the quotient would be non-Hausdorff (since two distinct points, $0$ and $x$, do not have disjoint neighborhoods).



              A Banach space, or even a normed space, is Hausdorff, as is every metric space.






              share|cite|improve this answer











              $endgroup$












              • $begingroup$
                Thanks! But it has been a long time since my last topology class, so would you give me some hint?
                $endgroup$
                – Zeng
                Nov 11 '18 at 18:39















              7












              $begingroup$

              If the subspace $K$ is not closed, then the quotient $X/K$ is not even Hausdorff, so does not meet the usual requirements of a topological vector space at all!



              (It's not about completeness or not, as $X/K$ is not metric in that case!)



              EDIT: In more detail, for $K$ not closed, and for $x$ in the closure of $K$ but not in $K$ itself, every neighborhood of $0$ in the quotient (with the quotient topology) contains $x$, but $xnot=0$ in the quotient. So the quotient would be non-Hausdorff (since two distinct points, $0$ and $x$, do not have disjoint neighborhoods).



              A Banach space, or even a normed space, is Hausdorff, as is every metric space.






              share|cite|improve this answer











              $endgroup$












              • $begingroup$
                Thanks! But it has been a long time since my last topology class, so would you give me some hint?
                $endgroup$
                – Zeng
                Nov 11 '18 at 18:39













              7












              7








              7





              $begingroup$

              If the subspace $K$ is not closed, then the quotient $X/K$ is not even Hausdorff, so does not meet the usual requirements of a topological vector space at all!



              (It's not about completeness or not, as $X/K$ is not metric in that case!)



              EDIT: In more detail, for $K$ not closed, and for $x$ in the closure of $K$ but not in $K$ itself, every neighborhood of $0$ in the quotient (with the quotient topology) contains $x$, but $xnot=0$ in the quotient. So the quotient would be non-Hausdorff (since two distinct points, $0$ and $x$, do not have disjoint neighborhoods).



              A Banach space, or even a normed space, is Hausdorff, as is every metric space.






              share|cite|improve this answer











              $endgroup$



              If the subspace $K$ is not closed, then the quotient $X/K$ is not even Hausdorff, so does not meet the usual requirements of a topological vector space at all!



              (It's not about completeness or not, as $X/K$ is not metric in that case!)



              EDIT: In more detail, for $K$ not closed, and for $x$ in the closure of $K$ but not in $K$ itself, every neighborhood of $0$ in the quotient (with the quotient topology) contains $x$, but $xnot=0$ in the quotient. So the quotient would be non-Hausdorff (since two distinct points, $0$ and $x$, do not have disjoint neighborhoods).



              A Banach space, or even a normed space, is Hausdorff, as is every metric space.







              share|cite|improve this answer














              share|cite|improve this answer



              share|cite|improve this answer








              edited Nov 11 '18 at 18:55

























              answered Nov 11 '18 at 18:31









              paul garrettpaul garrett

              31.9k362118




              31.9k362118











              • $begingroup$
                Thanks! But it has been a long time since my last topology class, so would you give me some hint?
                $endgroup$
                – Zeng
                Nov 11 '18 at 18:39
















              • $begingroup$
                Thanks! But it has been a long time since my last topology class, so would you give me some hint?
                $endgroup$
                – Zeng
                Nov 11 '18 at 18:39















              $begingroup$
              Thanks! But it has been a long time since my last topology class, so would you give me some hint?
              $endgroup$
              – Zeng
              Nov 11 '18 at 18:39




              $begingroup$
              Thanks! But it has been a long time since my last topology class, so would you give me some hint?
              $endgroup$
              – Zeng
              Nov 11 '18 at 18:39











              4












              $begingroup$

              If $K$ is not closed, then the function $lVert x+KrVert=inf_yin K lVert x-yrVert$ is not a norm on the quotient space, but just a seminorm, because $lVert x+KrVert=0$ for all $xinoverline K$.






              share|cite|improve this answer









              $endgroup$

















                4












                $begingroup$

                If $K$ is not closed, then the function $lVert x+KrVert=inf_yin K lVert x-yrVert$ is not a norm on the quotient space, but just a seminorm, because $lVert x+KrVert=0$ for all $xinoverline K$.






                share|cite|improve this answer









                $endgroup$















                  4












                  4








                  4





                  $begingroup$

                  If $K$ is not closed, then the function $lVert x+KrVert=inf_yin K lVert x-yrVert$ is not a norm on the quotient space, but just a seminorm, because $lVert x+KrVert=0$ for all $xinoverline K$.






                  share|cite|improve this answer









                  $endgroup$



                  If $K$ is not closed, then the function $lVert x+KrVert=inf_yin K lVert x-yrVert$ is not a norm on the quotient space, but just a seminorm, because $lVert x+KrVert=0$ for all $xinoverline K$.







                  share|cite|improve this answer












                  share|cite|improve this answer



                  share|cite|improve this answer










                  answered Nov 11 '18 at 18:32









                  Saucy O'PathSaucy O'Path

                  5,9591627




                  5,9591627





















                      2












                      $begingroup$

                      You do need closure of $K$. In particular, you need it to show that $X/K$ is in fact a normed space: If $| x + K |=0$, closure of $K$ implies that $x+K=K=0_X/K$.






                      share|cite|improve this answer









                      $endgroup$

















                        2












                        $begingroup$

                        You do need closure of $K$. In particular, you need it to show that $X/K$ is in fact a normed space: If $| x + K |=0$, closure of $K$ implies that $x+K=K=0_X/K$.






                        share|cite|improve this answer









                        $endgroup$















                          2












                          2








                          2





                          $begingroup$

                          You do need closure of $K$. In particular, you need it to show that $X/K$ is in fact a normed space: If $| x + K |=0$, closure of $K$ implies that $x+K=K=0_X/K$.






                          share|cite|improve this answer









                          $endgroup$



                          You do need closure of $K$. In particular, you need it to show that $X/K$ is in fact a normed space: If $| x + K |=0$, closure of $K$ implies that $x+K=K=0_X/K$.







                          share|cite|improve this answer












                          share|cite|improve this answer



                          share|cite|improve this answer










                          answered Nov 11 '18 at 18:36









                          Alonso DelfínAlonso Delfín

                          3,79411132




                          3,79411132



























                              draft saved

                              draft discarded
















































                              Thanks for contributing an answer to Mathematics Stack Exchange!


                              • Please be sure to answer the question. Provide details and share your research!

                              But avoid


                              • Asking for help, clarification, or responding to other answers.

                              • Making statements based on opinion; back them up with references or personal experience.

                              Use MathJax to format equations. MathJax reference.


                              To learn more, see our tips on writing great answers.




                              draft saved


                              draft discarded














                              StackExchange.ready(
                              function ()
                              StackExchange.openid.initPostLogin('.new-post-login', 'https%3a%2f%2fmath.stackexchange.com%2fquestions%2f2994221%2fwhy-should-k-be-closed-to-ensure-x-k-is-complete%23new-answer', 'question_page');

                              );

                              Post as a guest















                              Required, but never shown





















































                              Required, but never shown














                              Required, but never shown












                              Required, but never shown







                              Required, but never shown

































                              Required, but never shown














                              Required, but never shown












                              Required, but never shown







                              Required, but never shown







                              Popular posts from this blog

                              𛂒𛀶,𛀽𛀑𛂀𛃧𛂓𛀙𛃆𛃑𛃷𛂟𛁡𛀢𛀟𛁤𛂽𛁕𛁪𛂟𛂯,𛁞𛂧𛀴𛁄𛁠𛁼𛂿𛀤 𛂘,𛁺𛂾𛃭𛃭𛃵𛀺,𛂣𛃍𛂖𛃶 𛀸𛃀𛂖𛁶𛁏𛁚 𛂢𛂞 𛁰𛂆𛀔,𛁸𛀽𛁓𛃋𛂇𛃧𛀧𛃣𛂐𛃇,𛂂𛃻𛃲𛁬𛃞𛀧𛃃𛀅 𛂭𛁠𛁡𛃇𛀷𛃓𛁥,𛁙𛁘𛁞𛃸𛁸𛃣𛁜,𛂛,𛃿,𛁯𛂘𛂌𛃛𛁱𛃌𛂈𛂇 𛁊𛃲,𛀕𛃴𛀜 𛀶𛂆𛀶𛃟𛂉𛀣,𛂐𛁞𛁾 𛁷𛂑𛁳𛂯𛀬𛃅,𛃶𛁼

                              Edmonton

                              Crossroads (UK TV series)